Slot System
Featured Buckets
Featured Buckets Admin

Myth of the Month: NPO good for people with pancreatitis?

Article Type
Changed
Fri, 01/18/2019 - 15:55
Display Headline
Myth of the Month: NPO good for people with pancreatitis?

A 60-year-old man presents to the emergency department with nausea and abdominal pain, and is admitted with pancreatitis due to alcohol. In the evening after receiving pain medication, his abdominal pain is diminished but still present. He has an appetite and asks for food.

What do you recommend?

A. Nil per os (NPO) until pain is resolved.

B. NPO until amylase/lipase have normalized.

C. Nasogastric tube placement.

D. Okay to start feeding.

Myth: Treatment of pancreatitis includes early avoidance of food.

 

Dr. Douglas S. Paauw
Dr. Douglas S. Paauw

The conventional management of acute pancreatitis involves an NPO regimen until the pain and nausea have resolved.1 This dogma is offered because of the concern that food intake will stimulate pancreatic enzyme release in an already inflamed/injured pancreas.

The approach of NPO and slowly reintroducing feeding after prolonged periods of being without food is associated with pain relapses and increased length of hospitalizations.2 Nasojejunal feedings have become well accepted in patients with severe pancreatitis requiring ICU care.3

Are there data to show that oral feeding of patients with mild pancreatitis causes worse outcomes?

Dr. Niels Teich and colleagues randomized 143 hospitalized patients with mild pancreatitis to eating when they felt ready to (69 patients) vs. a group that were kept NPO until lipase levels returned to normal.4 The patients who started eating when they were ready left the hospital a day earlier than the patients who were fed only when lipase levels normalized (7 days vs. 8 days). There was no difference in abdominal pain between the two groups.

Dr. Maxim Petrov and colleagues looked at whether nasogastric tube feeding was preferable to NPO in patients with mild to moderate pancreatitis.5 In a randomized trial of 35 patients with pancreatitis, 17 received nasogastric feedings within 24 hours of admission, and 18 were NPO. The patients who received early nasogastric feedings had lower pain scores at 72 hours, compared with the NPO group (1 vs. 3 on a visual analog 10-point scale, P = .036). The number of patients who did not require opiates at 48 hours was also significantly less in the nasogastric feeding group (9 vs. 3, P = .024).

I think the most striking difference was in patients’ ability to tolerate oral feeding. Patients in both groups received oral food at an average of 4 days; only 1 of 17 patients in the nasogastric feeding group could not tolerate feeding, compared with 9 of 18 patients in the NPO group.

Dr. Gunilla Eckerwall and colleagues studied the outcome of immediate oral feeding in patients with mild pancreatitis.6 Sixty patients with mild acute pancreatitis, defined by amylase greater than 3 times normal and APACHE scores less than 8, were randomized to either immediate oral feeding (30 patients) or fasting (30 patients). Key outcome measures in the study were amylase, systemic inflammatory response, and length of hospital stay.

There were no differences in amylase levels, labs measuring systemic inflammatory response, or gastrointestinal symptoms between the two groups. The immediate oral feeding group had a significantly shorter length of hospital stay than the fasting group (4 days vs. 6 days, P less than .05).

So, what does all this tell us about feeding patients with acute pancreatitis? For mild to moderate acute pancreatitis, the outcomes appear to be no worse when patients are fed early. There may be a trend to quicker hospital discharge in those who get fed earlier. The studies have all been small, and a large multicenter trial would be welcome.

References

1. Gastroenterology. 2007 May;132(5):2022-44.

2. Gut. 1997 Feb;40(2):262-6.

3. Am J Gastroenterol. 2006 Oct;101(10):2379-400.

4. Pancreas. 2010 Oct;39(7):1088-92.

5. Clin Nutr. 2013 Oct;32(5):697-703.

6. Clin Nutr. 2007 Dec;26(6):758-63.

Dr. Paauw is professor of medicine in the division of general internal medicine at the University of Washington, Seattle, and he serves as third-year medical student clerkship director at the University of Washington. Contact Dr. Paauw at dpaauw@uw.edu.

Publications
Topics
Legacy Keywords
acute pancreatitis
Sections

A 60-year-old man presents to the emergency department with nausea and abdominal pain, and is admitted with pancreatitis due to alcohol. In the evening after receiving pain medication, his abdominal pain is diminished but still present. He has an appetite and asks for food.

What do you recommend?

A. Nil per os (NPO) until pain is resolved.

B. NPO until amylase/lipase have normalized.

C. Nasogastric tube placement.

D. Okay to start feeding.

Myth: Treatment of pancreatitis includes early avoidance of food.

 

Dr. Douglas S. Paauw
Dr. Douglas S. Paauw

The conventional management of acute pancreatitis involves an NPO regimen until the pain and nausea have resolved.1 This dogma is offered because of the concern that food intake will stimulate pancreatic enzyme release in an already inflamed/injured pancreas.

The approach of NPO and slowly reintroducing feeding after prolonged periods of being without food is associated with pain relapses and increased length of hospitalizations.2 Nasojejunal feedings have become well accepted in patients with severe pancreatitis requiring ICU care.3

Are there data to show that oral feeding of patients with mild pancreatitis causes worse outcomes?

Dr. Niels Teich and colleagues randomized 143 hospitalized patients with mild pancreatitis to eating when they felt ready to (69 patients) vs. a group that were kept NPO until lipase levels returned to normal.4 The patients who started eating when they were ready left the hospital a day earlier than the patients who were fed only when lipase levels normalized (7 days vs. 8 days). There was no difference in abdominal pain between the two groups.

Dr. Maxim Petrov and colleagues looked at whether nasogastric tube feeding was preferable to NPO in patients with mild to moderate pancreatitis.5 In a randomized trial of 35 patients with pancreatitis, 17 received nasogastric feedings within 24 hours of admission, and 18 were NPO. The patients who received early nasogastric feedings had lower pain scores at 72 hours, compared with the NPO group (1 vs. 3 on a visual analog 10-point scale, P = .036). The number of patients who did not require opiates at 48 hours was also significantly less in the nasogastric feeding group (9 vs. 3, P = .024).

I think the most striking difference was in patients’ ability to tolerate oral feeding. Patients in both groups received oral food at an average of 4 days; only 1 of 17 patients in the nasogastric feeding group could not tolerate feeding, compared with 9 of 18 patients in the NPO group.

Dr. Gunilla Eckerwall and colleagues studied the outcome of immediate oral feeding in patients with mild pancreatitis.6 Sixty patients with mild acute pancreatitis, defined by amylase greater than 3 times normal and APACHE scores less than 8, were randomized to either immediate oral feeding (30 patients) or fasting (30 patients). Key outcome measures in the study were amylase, systemic inflammatory response, and length of hospital stay.

There were no differences in amylase levels, labs measuring systemic inflammatory response, or gastrointestinal symptoms between the two groups. The immediate oral feeding group had a significantly shorter length of hospital stay than the fasting group (4 days vs. 6 days, P less than .05).

So, what does all this tell us about feeding patients with acute pancreatitis? For mild to moderate acute pancreatitis, the outcomes appear to be no worse when patients are fed early. There may be a trend to quicker hospital discharge in those who get fed earlier. The studies have all been small, and a large multicenter trial would be welcome.

References

1. Gastroenterology. 2007 May;132(5):2022-44.

2. Gut. 1997 Feb;40(2):262-6.

3. Am J Gastroenterol. 2006 Oct;101(10):2379-400.

4. Pancreas. 2010 Oct;39(7):1088-92.

5. Clin Nutr. 2013 Oct;32(5):697-703.

6. Clin Nutr. 2007 Dec;26(6):758-63.

Dr. Paauw is professor of medicine in the division of general internal medicine at the University of Washington, Seattle, and he serves as third-year medical student clerkship director at the University of Washington. Contact Dr. Paauw at dpaauw@uw.edu.

A 60-year-old man presents to the emergency department with nausea and abdominal pain, and is admitted with pancreatitis due to alcohol. In the evening after receiving pain medication, his abdominal pain is diminished but still present. He has an appetite and asks for food.

What do you recommend?

A. Nil per os (NPO) until pain is resolved.

B. NPO until amylase/lipase have normalized.

C. Nasogastric tube placement.

D. Okay to start feeding.

Myth: Treatment of pancreatitis includes early avoidance of food.

 

Dr. Douglas S. Paauw
Dr. Douglas S. Paauw

The conventional management of acute pancreatitis involves an NPO regimen until the pain and nausea have resolved.1 This dogma is offered because of the concern that food intake will stimulate pancreatic enzyme release in an already inflamed/injured pancreas.

The approach of NPO and slowly reintroducing feeding after prolonged periods of being without food is associated with pain relapses and increased length of hospitalizations.2 Nasojejunal feedings have become well accepted in patients with severe pancreatitis requiring ICU care.3

Are there data to show that oral feeding of patients with mild pancreatitis causes worse outcomes?

Dr. Niels Teich and colleagues randomized 143 hospitalized patients with mild pancreatitis to eating when they felt ready to (69 patients) vs. a group that were kept NPO until lipase levels returned to normal.4 The patients who started eating when they were ready left the hospital a day earlier than the patients who were fed only when lipase levels normalized (7 days vs. 8 days). There was no difference in abdominal pain between the two groups.

Dr. Maxim Petrov and colleagues looked at whether nasogastric tube feeding was preferable to NPO in patients with mild to moderate pancreatitis.5 In a randomized trial of 35 patients with pancreatitis, 17 received nasogastric feedings within 24 hours of admission, and 18 were NPO. The patients who received early nasogastric feedings had lower pain scores at 72 hours, compared with the NPO group (1 vs. 3 on a visual analog 10-point scale, P = .036). The number of patients who did not require opiates at 48 hours was also significantly less in the nasogastric feeding group (9 vs. 3, P = .024).

I think the most striking difference was in patients’ ability to tolerate oral feeding. Patients in both groups received oral food at an average of 4 days; only 1 of 17 patients in the nasogastric feeding group could not tolerate feeding, compared with 9 of 18 patients in the NPO group.

Dr. Gunilla Eckerwall and colleagues studied the outcome of immediate oral feeding in patients with mild pancreatitis.6 Sixty patients with mild acute pancreatitis, defined by amylase greater than 3 times normal and APACHE scores less than 8, were randomized to either immediate oral feeding (30 patients) or fasting (30 patients). Key outcome measures in the study were amylase, systemic inflammatory response, and length of hospital stay.

There were no differences in amylase levels, labs measuring systemic inflammatory response, or gastrointestinal symptoms between the two groups. The immediate oral feeding group had a significantly shorter length of hospital stay than the fasting group (4 days vs. 6 days, P less than .05).

So, what does all this tell us about feeding patients with acute pancreatitis? For mild to moderate acute pancreatitis, the outcomes appear to be no worse when patients are fed early. There may be a trend to quicker hospital discharge in those who get fed earlier. The studies have all been small, and a large multicenter trial would be welcome.

References

1. Gastroenterology. 2007 May;132(5):2022-44.

2. Gut. 1997 Feb;40(2):262-6.

3. Am J Gastroenterol. 2006 Oct;101(10):2379-400.

4. Pancreas. 2010 Oct;39(7):1088-92.

5. Clin Nutr. 2013 Oct;32(5):697-703.

6. Clin Nutr. 2007 Dec;26(6):758-63.

Dr. Paauw is professor of medicine in the division of general internal medicine at the University of Washington, Seattle, and he serves as third-year medical student clerkship director at the University of Washington. Contact Dr. Paauw at dpaauw@uw.edu.

Publications
Publications
Topics
Article Type
Display Headline
Myth of the Month: NPO good for people with pancreatitis?
Display Headline
Myth of the Month: NPO good for people with pancreatitis?
Legacy Keywords
acute pancreatitis
Legacy Keywords
acute pancreatitis
Sections
Disallow All Ads

Myth of the Month: Retired myths from yesteryear

Article Type
Changed
Fri, 01/18/2019 - 15:52
Display Headline
Myth of the Month: Retired myths from yesteryear

I appreciate the opportunity to share medical myths each month, hopefully to highlight topics where new knowledge and data may help change ways we approach common problems in medicine. I have been researching medical myths since the early 1990s, and some have actually evolved in such a way that they are no longer myths – that is, accepted practice now is very different than it was decades ago, and has incorporated updated research.

Some myths are timeless. The vitamin B12 myth I shared in this column last year continues to this day, despite evidence that has been present since the 1960s.

 

Dr. Douglas S. Paauw
Dr. Douglas S. Paauw

I will share with you two of my all-time favorite myths that have now been retired, where current practice now does not perpetuate these myths.

When I was in medical school, I was taught that the best way to treat a corneal abrasion was to patch the affected eye.1 Pretty much everyone who was seen in an emergency department for a corneal abrasion before the 1990s left the ED with an eye patch. This standard approach was not based on any evidence of benefit of healing or decreased pain.

Dr. Harold Jackson reported in a study of patients with corneal abrasions published in 1960 that there was no difference in healing between eyes that were patched and eyes that were left unpatched.2 The largest published study on eye patches for corneal abrasions involved 201 patients who were evaluated for corneal abrasions.3 The patients who did not receive an eye patch had less pain and quicker healing of the corneal abrasions. Other studies all showed no benefit to eye patches.4,5

A Cochrane Review published in 2006 concluded: “Treating simple corneal abrasions with a patch does not improve healing rates on the first day post-injury and does not reduce pain. In addition, use of patches results in a loss of binocular vision. Therefore, it is recommended that patches should not be used for simple corneal abrasions.”6

A more recent study by Dr. Moreno Menghini and colleagues showed no differences in healing of traumatic corneal abrasions between groups who received an eye patch, a contact lens, or no eye covering.7

Another longstanding myth that is less commonly seen now is the avoidance of use of narcotics for the treatment of acute, severe abdominal pain.

The long-term teaching was that by treating abdominal pain with narcotics, you could mask the important physical exam findings in patients presenting with an acute abdomen. The source of this myth wasn’t hard to uncover. The following are quotes from Cope’s Early Diagnosis of the Acute Abdomen 15th and 16th editions (these were the editions available back when I was a medical student in the early 1980s).

From the 15th edition: “If morphine be given, it is possible for a patient to die happy in the belief that he is on the road to recovery, and in some cases the medical attendant may for a time be induced to share the elusive hope.”8

An even stronger position was taken in the next edition of Cope’s text: “The patient cried out for relief, the relatives are insistent that something should be done, and the humane disciple of Aesculapius may think it is his first duty to diminish or banish the too obvious agony by administering a narcotic. Such a policy is a mistake. Though it may appear cruel, it is really kind to withhold morphine until a reasonable diagnosis has been made.”9

No controlled trials ever questioned this long-held belief until a study done by Dr. Alex Attard and colleagues published in 1992.10 In this study, 100 patients were evaluated by an admitting officer and given an intramuscular injection of either a narcotic or saline. Surgeons who subsequently followed the patients felt equally confident in diagnosis and management in both groups. The decision to operate or observe was incorrect in two patients in the narcotic group and nine in the saline group.

Dr. H. A. Amoli and colleagues studied whether administering morphine changed exam findings in patients with acute appendiciits.11 In a randomized, double-blind study design, half the patients received morphine and half received saline. Patients were examined by surgeons not involved in their care before and after drug administration, and their pain intensity and signs were recorded at each visit. The administration of morphine did not alter clinical signs or physician management plans.

In a study by Dr. Steven Pace and colleagues of patients presenting with acute abdominal pain, intravenous morphine or placebo was administered in 71 patients early in their presentation to the ED.12 There were no differences in accuracy of diagnosis between groups. Three diagnostic or management errors were made in each group.

 

 

I think the standard of care now for corneal abrasion treatment does not include eye patching. I also believe that the old teaching of no pain medication until the surgeon has examined the patient has also been replaced with appropriate pain management occurring early in the care plan for patients presenting with acute abdominal pain.

In the case of corneal abrasions, overwhelming data showing no benefit won out. I believe that the change in the management of acute abdominal pain was a combination of data along with advances in diagnostic imaging.

References

1. Wilkins. Emergency Medicine. 1989 Williams and Wilkins, Baltimore, Md.

2. Br Med J. 1960 Sep 3;2(5200):713.

3. Ophthalmology. 1995 Dec;102(12):1936-42.

4. Lancet. 1991 Mar 16;337(8742):643.

5. Eye (Lond). 1993;7:468-71.

6. Cochrane Database Syst Rev. 2006 Apr 19;(2):CD004764.

7. Ophthalmic Res. 2013;50(1):13-8.

8. Cope’s Early Diagnosis of the Acute Abdomen, 15th Edition, Oxford University Press, 1979.

9. Cope’s Early Diagnosis of the Acute Abdomen, 16th Edition, Oxford University Press, 1983.

10. BMJ. 1992 Sep 5;305(6853):554-6.

11. Emerg Med J. 2008 Sep;25(9):586-9.

12. Acad Emerg Med. 1996 Dec;3(12):1086-92.

Dr. Paauw is professor of medicine in the division of general internal medicine at the University of Washington, Seattle, and he serves as third-year medical student clerkship director at the University of Washington. Contact Dr. Paauw at dpaauw@uw.edu.

Publications
Topics
Legacy Keywords
corneal abrasion, abdominal pain, appendicitis, eye injury
Sections

I appreciate the opportunity to share medical myths each month, hopefully to highlight topics where new knowledge and data may help change ways we approach common problems in medicine. I have been researching medical myths since the early 1990s, and some have actually evolved in such a way that they are no longer myths – that is, accepted practice now is very different than it was decades ago, and has incorporated updated research.

Some myths are timeless. The vitamin B12 myth I shared in this column last year continues to this day, despite evidence that has been present since the 1960s.

 

Dr. Douglas S. Paauw
Dr. Douglas S. Paauw

I will share with you two of my all-time favorite myths that have now been retired, where current practice now does not perpetuate these myths.

When I was in medical school, I was taught that the best way to treat a corneal abrasion was to patch the affected eye.1 Pretty much everyone who was seen in an emergency department for a corneal abrasion before the 1990s left the ED with an eye patch. This standard approach was not based on any evidence of benefit of healing or decreased pain.

Dr. Harold Jackson reported in a study of patients with corneal abrasions published in 1960 that there was no difference in healing between eyes that were patched and eyes that were left unpatched.2 The largest published study on eye patches for corneal abrasions involved 201 patients who were evaluated for corneal abrasions.3 The patients who did not receive an eye patch had less pain and quicker healing of the corneal abrasions. Other studies all showed no benefit to eye patches.4,5

A Cochrane Review published in 2006 concluded: “Treating simple corneal abrasions with a patch does not improve healing rates on the first day post-injury and does not reduce pain. In addition, use of patches results in a loss of binocular vision. Therefore, it is recommended that patches should not be used for simple corneal abrasions.”6

A more recent study by Dr. Moreno Menghini and colleagues showed no differences in healing of traumatic corneal abrasions between groups who received an eye patch, a contact lens, or no eye covering.7

Another longstanding myth that is less commonly seen now is the avoidance of use of narcotics for the treatment of acute, severe abdominal pain.

The long-term teaching was that by treating abdominal pain with narcotics, you could mask the important physical exam findings in patients presenting with an acute abdomen. The source of this myth wasn’t hard to uncover. The following are quotes from Cope’s Early Diagnosis of the Acute Abdomen 15th and 16th editions (these were the editions available back when I was a medical student in the early 1980s).

From the 15th edition: “If morphine be given, it is possible for a patient to die happy in the belief that he is on the road to recovery, and in some cases the medical attendant may for a time be induced to share the elusive hope.”8

An even stronger position was taken in the next edition of Cope’s text: “The patient cried out for relief, the relatives are insistent that something should be done, and the humane disciple of Aesculapius may think it is his first duty to diminish or banish the too obvious agony by administering a narcotic. Such a policy is a mistake. Though it may appear cruel, it is really kind to withhold morphine until a reasonable diagnosis has been made.”9

No controlled trials ever questioned this long-held belief until a study done by Dr. Alex Attard and colleagues published in 1992.10 In this study, 100 patients were evaluated by an admitting officer and given an intramuscular injection of either a narcotic or saline. Surgeons who subsequently followed the patients felt equally confident in diagnosis and management in both groups. The decision to operate or observe was incorrect in two patients in the narcotic group and nine in the saline group.

Dr. H. A. Amoli and colleagues studied whether administering morphine changed exam findings in patients with acute appendiciits.11 In a randomized, double-blind study design, half the patients received morphine and half received saline. Patients were examined by surgeons not involved in their care before and after drug administration, and their pain intensity and signs were recorded at each visit. The administration of morphine did not alter clinical signs or physician management plans.

In a study by Dr. Steven Pace and colleagues of patients presenting with acute abdominal pain, intravenous morphine or placebo was administered in 71 patients early in their presentation to the ED.12 There were no differences in accuracy of diagnosis between groups. Three diagnostic or management errors were made in each group.

 

 

I think the standard of care now for corneal abrasion treatment does not include eye patching. I also believe that the old teaching of no pain medication until the surgeon has examined the patient has also been replaced with appropriate pain management occurring early in the care plan for patients presenting with acute abdominal pain.

In the case of corneal abrasions, overwhelming data showing no benefit won out. I believe that the change in the management of acute abdominal pain was a combination of data along with advances in diagnostic imaging.

References

1. Wilkins. Emergency Medicine. 1989 Williams and Wilkins, Baltimore, Md.

2. Br Med J. 1960 Sep 3;2(5200):713.

3. Ophthalmology. 1995 Dec;102(12):1936-42.

4. Lancet. 1991 Mar 16;337(8742):643.

5. Eye (Lond). 1993;7:468-71.

6. Cochrane Database Syst Rev. 2006 Apr 19;(2):CD004764.

7. Ophthalmic Res. 2013;50(1):13-8.

8. Cope’s Early Diagnosis of the Acute Abdomen, 15th Edition, Oxford University Press, 1979.

9. Cope’s Early Diagnosis of the Acute Abdomen, 16th Edition, Oxford University Press, 1983.

10. BMJ. 1992 Sep 5;305(6853):554-6.

11. Emerg Med J. 2008 Sep;25(9):586-9.

12. Acad Emerg Med. 1996 Dec;3(12):1086-92.

Dr. Paauw is professor of medicine in the division of general internal medicine at the University of Washington, Seattle, and he serves as third-year medical student clerkship director at the University of Washington. Contact Dr. Paauw at dpaauw@uw.edu.

I appreciate the opportunity to share medical myths each month, hopefully to highlight topics where new knowledge and data may help change ways we approach common problems in medicine. I have been researching medical myths since the early 1990s, and some have actually evolved in such a way that they are no longer myths – that is, accepted practice now is very different than it was decades ago, and has incorporated updated research.

Some myths are timeless. The vitamin B12 myth I shared in this column last year continues to this day, despite evidence that has been present since the 1960s.

 

Dr. Douglas S. Paauw
Dr. Douglas S. Paauw

I will share with you two of my all-time favorite myths that have now been retired, where current practice now does not perpetuate these myths.

When I was in medical school, I was taught that the best way to treat a corneal abrasion was to patch the affected eye.1 Pretty much everyone who was seen in an emergency department for a corneal abrasion before the 1990s left the ED with an eye patch. This standard approach was not based on any evidence of benefit of healing or decreased pain.

Dr. Harold Jackson reported in a study of patients with corneal abrasions published in 1960 that there was no difference in healing between eyes that were patched and eyes that were left unpatched.2 The largest published study on eye patches for corneal abrasions involved 201 patients who were evaluated for corneal abrasions.3 The patients who did not receive an eye patch had less pain and quicker healing of the corneal abrasions. Other studies all showed no benefit to eye patches.4,5

A Cochrane Review published in 2006 concluded: “Treating simple corneal abrasions with a patch does not improve healing rates on the first day post-injury and does not reduce pain. In addition, use of patches results in a loss of binocular vision. Therefore, it is recommended that patches should not be used for simple corneal abrasions.”6

A more recent study by Dr. Moreno Menghini and colleagues showed no differences in healing of traumatic corneal abrasions between groups who received an eye patch, a contact lens, or no eye covering.7

Another longstanding myth that is less commonly seen now is the avoidance of use of narcotics for the treatment of acute, severe abdominal pain.

The long-term teaching was that by treating abdominal pain with narcotics, you could mask the important physical exam findings in patients presenting with an acute abdomen. The source of this myth wasn’t hard to uncover. The following are quotes from Cope’s Early Diagnosis of the Acute Abdomen 15th and 16th editions (these were the editions available back when I was a medical student in the early 1980s).

From the 15th edition: “If morphine be given, it is possible for a patient to die happy in the belief that he is on the road to recovery, and in some cases the medical attendant may for a time be induced to share the elusive hope.”8

An even stronger position was taken in the next edition of Cope’s text: “The patient cried out for relief, the relatives are insistent that something should be done, and the humane disciple of Aesculapius may think it is his first duty to diminish or banish the too obvious agony by administering a narcotic. Such a policy is a mistake. Though it may appear cruel, it is really kind to withhold morphine until a reasonable diagnosis has been made.”9

No controlled trials ever questioned this long-held belief until a study done by Dr. Alex Attard and colleagues published in 1992.10 In this study, 100 patients were evaluated by an admitting officer and given an intramuscular injection of either a narcotic or saline. Surgeons who subsequently followed the patients felt equally confident in diagnosis and management in both groups. The decision to operate or observe was incorrect in two patients in the narcotic group and nine in the saline group.

Dr. H. A. Amoli and colleagues studied whether administering morphine changed exam findings in patients with acute appendiciits.11 In a randomized, double-blind study design, half the patients received morphine and half received saline. Patients were examined by surgeons not involved in their care before and after drug administration, and their pain intensity and signs were recorded at each visit. The administration of morphine did not alter clinical signs or physician management plans.

In a study by Dr. Steven Pace and colleagues of patients presenting with acute abdominal pain, intravenous morphine or placebo was administered in 71 patients early in their presentation to the ED.12 There were no differences in accuracy of diagnosis between groups. Three diagnostic or management errors were made in each group.

 

 

I think the standard of care now for corneal abrasion treatment does not include eye patching. I also believe that the old teaching of no pain medication until the surgeon has examined the patient has also been replaced with appropriate pain management occurring early in the care plan for patients presenting with acute abdominal pain.

In the case of corneal abrasions, overwhelming data showing no benefit won out. I believe that the change in the management of acute abdominal pain was a combination of data along with advances in diagnostic imaging.

References

1. Wilkins. Emergency Medicine. 1989 Williams and Wilkins, Baltimore, Md.

2. Br Med J. 1960 Sep 3;2(5200):713.

3. Ophthalmology. 1995 Dec;102(12):1936-42.

4. Lancet. 1991 Mar 16;337(8742):643.

5. Eye (Lond). 1993;7:468-71.

6. Cochrane Database Syst Rev. 2006 Apr 19;(2):CD004764.

7. Ophthalmic Res. 2013;50(1):13-8.

8. Cope’s Early Diagnosis of the Acute Abdomen, 15th Edition, Oxford University Press, 1979.

9. Cope’s Early Diagnosis of the Acute Abdomen, 16th Edition, Oxford University Press, 1983.

10. BMJ. 1992 Sep 5;305(6853):554-6.

11. Emerg Med J. 2008 Sep;25(9):586-9.

12. Acad Emerg Med. 1996 Dec;3(12):1086-92.

Dr. Paauw is professor of medicine in the division of general internal medicine at the University of Washington, Seattle, and he serves as third-year medical student clerkship director at the University of Washington. Contact Dr. Paauw at dpaauw@uw.edu.

Publications
Publications
Topics
Article Type
Display Headline
Myth of the Month: Retired myths from yesteryear
Display Headline
Myth of the Month: Retired myths from yesteryear
Legacy Keywords
corneal abrasion, abdominal pain, appendicitis, eye injury
Legacy Keywords
corneal abrasion, abdominal pain, appendicitis, eye injury
Sections
Disallow All Ads

Drug interaction myths

Article Type
Changed
Fri, 01/18/2019 - 15:47
Display Headline
Drug interaction myths

A 72-year-old man with benign prostatic hypertrophy comes to clinic to discuss recent problems with erectile dysfunction. He has been treated with tamsulosin with good results for the past 3 years for his BPH. He is given a prescription for vardenafil 10 mg for his ED. The pharmacist calls and asks if you want the prescription filled despite a drug interaction. What do you recommend?

A. Fill the prescription as written.

B. Have the patient take half a tablet of vardenafil.

C. Have the patient not take vardenafil within 6 hours of taking tamsulosin.

A 22-year-old woman presents with a unilateral headache, pounding in nature, worse with exercise. She is diagnosed with migraine. She has a history of depression and is taking 40 mg of fluoxetine. She is given a prescription for sumatriptan 100 mg. The pharmacist calls you and asks if you want to make changes because of possible drug interaction. What do you recommend?

A. Fill the prescription as written.

B. Have the patient take 50 mg of sumatriptan.

C. Have her reduce her fluoxetine dose to 20 mg.

D. Do not take sumatriptan within 12 hours of taking fluoxetine.

The title of this article is drug interaction myths. These are not true myths, but in both these cases, I think the prescriptions should be filled as written, and it will be safe for the patient to take the medications despite a theoretical drug interaction.

I have received calls from the pharmacist multiple times when I have prescribed these drug combinations, and I will share with you the evidence of safety for using these medications despite potential interactions.

In 2006, the Food and Drug Administration released an alert on serotonin syndrome occurring with combined use of selective serotonin reuptake inhibitors (SSRIs) or serotonin-norepinephrine reuptake inhibitors (SNRIs) with triptans.1 This alert was based on 29 cases that the FDA evaluated and felt justified an alert.

Dr. Randolph W. Evans did an analysis of all 29 cases to see if they met criteria for serotonin syndrome.2 He classified if the cases met two different criteria for serotonin syndrome: the Hunter criteria3 or the Sternbach criteria4.

Of the 29 case reports, 10 met the Sternbach criteria, and none of the reports met the Hunter criteria. Some of the cases included polypharmacy of other drugs that can cause serotonin syndrome. Two cases that met the Sternbach criteria were excluded because they were either not on an SSRI or had alternative compelling diagnoses.

Dr. Evans suggested the biologic implausibility of triptans causing serotonin syndrome, because serotonin syndrome is believed to be caused by activation of 5-HT1A and 5-HT2A receptors, whereas triptans act at the 5-HT1B/5-HT1D and 5-HT1F receptors.

In a prospective study of 12,339 patients with migraine who used subcutaneous sumatriptan for at least 1 year, 1,784 patients also received an SSRI.5 No episodes of serotonin syndrome were reported. David A. Sclar, Ph.D., and his colleagues estimated that in 2007-2008, 1.4 million patients were prescribed both a triptan and an SSRI or SNRI.6 That is a 36% increase from 2003-2004, despite a 50% reduction in coprescriptions from primary care physicians – suggesting neurologists were not affected by the FDA alert.7

The American Headache Society position paper on the FDA alert states, “The currently available evidence does not support limiting the use of triptans with SNRIs or SSRIs, or the use of triptan monotherapy, due to concerns for serotonin syndrome.”8

A warning will pop up on prescribing software when you prescribe a phosphodiesterase inhibitor in patients who are taking alpha-blockers. This is a common situation, because BPH and ED both become more common with age. The concern is that the combination of alpha-blocker plus phosphodiesterase inhibitor will increase the risk of hypotension.

Dr. Michel Guillaume and his colleagues studied the hemodynamic effect of doxazosin and tamsulosin in combination with tadalafil.9 A total of 45 healthy men aged 40-70 years were randomized to receive tadalafil and placebo for 28 days. Doxazosin was added after 7 days and continued for an additional 21 days. The second study included 39 men who received tadalafil and placebo for 7 days before adding tamsulosin for an additional 7 days.

There were no significant differences in change in standing systolic blood pressure with tadalafil with placebo, doxazosin, or tamsulosin.

Robert A. Kloner, M.D., Ph.D., and his colleagues reported on a randomized, double-blind, crossover trial of doxazosin 8 mg or placebo with tadalafil 20 mg and tamsulosin 0.4 mg or placebo with 10 mg or 20 mg of tadalafil.10 Tadalafil did augment the hypotensive effect of doxazosin, but it did not have any blood pressure effect on patients taking tamsulosin.

 

 

In a study of men taking both tamsulosin and vardenafil or tamsulosin and placebo for the treatment of BPH symptoms, Dr. Mauro Gacci and his colleagues found no significant difference in adverse effects in patients who received tamsulosin plus placebo, compared with men who received tamsulosin plus vardenafil.11

I think it is safe to prescribe triptans in patients who are on SSRIs and SNRIs. In patients who need both alpha-blockers and phosphodiesterase inhibitors, I think tamsulosin is the safest alpha-blocker option. It is best to not start a phosphodiesterase inhibitor at the same time as an alpha-blocker. The studies on coadministration of alpha-blockers and phosphodiesterase inhibitors have been done in either healthy volunteers, or in patients without severe systemic disease. So, the effect on blood pressure in patients taking multiple antihypertensive drugs or heart failure drugs is unknown.

References

1. U.S. Food and Drug Administration. Information for healthcare professionals: Selective serotonin reuptake inhibitors (SSRIs), selective serotonin-norepinephrine reuptake inhibitors (SNRIs), 5-hydroxytryptamine receptor agonists (triptans), July 19, 2006.

2. MedGenMed. 2007 Sep 5;9(3):48.

3. QJM. 2003 Sep;96(9):635-42.

4. Am J Psychiatry. 1991 Jun;148(6):705-13.

5. Cephalalgia. 1999 Sep;19(7):668-75.

6. Headache. 2012 Feb;52(2):198-203.

7. Headache. 2012 Feb;52(2):195-7.

8. Headache. 2010 Jun;50(6):1089-99.

9. J Clin Pharmacol. 2007 Oct;47(10):1303-10.

10. J Urol. 2004 Nov;172(5 Pt 1):1935-40.

11. J Sex Med. 2012 Jun;9(6):1624-33.

Dr. Paauw is professor of medicine in the division of general internal medicine at the University of Washington, Seattle, and he serves as third-year medical student clerkship director at the University of Washington. Contact Dr. Paauw at dpaauw@uw.edu.

Publications
Topics
Legacy Keywords
durg interactions, SSRI, SNRI, antidepressant, migraine, triptan, sumatriptan, PDE5 inhibitor, hypertension, BPH, alpha blocker, Levitra, Cialis, Viagra
Sections

A 72-year-old man with benign prostatic hypertrophy comes to clinic to discuss recent problems with erectile dysfunction. He has been treated with tamsulosin with good results for the past 3 years for his BPH. He is given a prescription for vardenafil 10 mg for his ED. The pharmacist calls and asks if you want the prescription filled despite a drug interaction. What do you recommend?

A. Fill the prescription as written.

B. Have the patient take half a tablet of vardenafil.

C. Have the patient not take vardenafil within 6 hours of taking tamsulosin.

A 22-year-old woman presents with a unilateral headache, pounding in nature, worse with exercise. She is diagnosed with migraine. She has a history of depression and is taking 40 mg of fluoxetine. She is given a prescription for sumatriptan 100 mg. The pharmacist calls you and asks if you want to make changes because of possible drug interaction. What do you recommend?

A. Fill the prescription as written.

B. Have the patient take 50 mg of sumatriptan.

C. Have her reduce her fluoxetine dose to 20 mg.

D. Do not take sumatriptan within 12 hours of taking fluoxetine.

The title of this article is drug interaction myths. These are not true myths, but in both these cases, I think the prescriptions should be filled as written, and it will be safe for the patient to take the medications despite a theoretical drug interaction.

I have received calls from the pharmacist multiple times when I have prescribed these drug combinations, and I will share with you the evidence of safety for using these medications despite potential interactions.

In 2006, the Food and Drug Administration released an alert on serotonin syndrome occurring with combined use of selective serotonin reuptake inhibitors (SSRIs) or serotonin-norepinephrine reuptake inhibitors (SNRIs) with triptans.1 This alert was based on 29 cases that the FDA evaluated and felt justified an alert.

Dr. Randolph W. Evans did an analysis of all 29 cases to see if they met criteria for serotonin syndrome.2 He classified if the cases met two different criteria for serotonin syndrome: the Hunter criteria3 or the Sternbach criteria4.

Of the 29 case reports, 10 met the Sternbach criteria, and none of the reports met the Hunter criteria. Some of the cases included polypharmacy of other drugs that can cause serotonin syndrome. Two cases that met the Sternbach criteria were excluded because they were either not on an SSRI or had alternative compelling diagnoses.

Dr. Evans suggested the biologic implausibility of triptans causing serotonin syndrome, because serotonin syndrome is believed to be caused by activation of 5-HT1A and 5-HT2A receptors, whereas triptans act at the 5-HT1B/5-HT1D and 5-HT1F receptors.

In a prospective study of 12,339 patients with migraine who used subcutaneous sumatriptan for at least 1 year, 1,784 patients also received an SSRI.5 No episodes of serotonin syndrome were reported. David A. Sclar, Ph.D., and his colleagues estimated that in 2007-2008, 1.4 million patients were prescribed both a triptan and an SSRI or SNRI.6 That is a 36% increase from 2003-2004, despite a 50% reduction in coprescriptions from primary care physicians – suggesting neurologists were not affected by the FDA alert.7

The American Headache Society position paper on the FDA alert states, “The currently available evidence does not support limiting the use of triptans with SNRIs or SSRIs, or the use of triptan monotherapy, due to concerns for serotonin syndrome.”8

A warning will pop up on prescribing software when you prescribe a phosphodiesterase inhibitor in patients who are taking alpha-blockers. This is a common situation, because BPH and ED both become more common with age. The concern is that the combination of alpha-blocker plus phosphodiesterase inhibitor will increase the risk of hypotension.

Dr. Michel Guillaume and his colleagues studied the hemodynamic effect of doxazosin and tamsulosin in combination with tadalafil.9 A total of 45 healthy men aged 40-70 years were randomized to receive tadalafil and placebo for 28 days. Doxazosin was added after 7 days and continued for an additional 21 days. The second study included 39 men who received tadalafil and placebo for 7 days before adding tamsulosin for an additional 7 days.

There were no significant differences in change in standing systolic blood pressure with tadalafil with placebo, doxazosin, or tamsulosin.

Robert A. Kloner, M.D., Ph.D., and his colleagues reported on a randomized, double-blind, crossover trial of doxazosin 8 mg or placebo with tadalafil 20 mg and tamsulosin 0.4 mg or placebo with 10 mg or 20 mg of tadalafil.10 Tadalafil did augment the hypotensive effect of doxazosin, but it did not have any blood pressure effect on patients taking tamsulosin.

 

 

In a study of men taking both tamsulosin and vardenafil or tamsulosin and placebo for the treatment of BPH symptoms, Dr. Mauro Gacci and his colleagues found no significant difference in adverse effects in patients who received tamsulosin plus placebo, compared with men who received tamsulosin plus vardenafil.11

I think it is safe to prescribe triptans in patients who are on SSRIs and SNRIs. In patients who need both alpha-blockers and phosphodiesterase inhibitors, I think tamsulosin is the safest alpha-blocker option. It is best to not start a phosphodiesterase inhibitor at the same time as an alpha-blocker. The studies on coadministration of alpha-blockers and phosphodiesterase inhibitors have been done in either healthy volunteers, or in patients without severe systemic disease. So, the effect on blood pressure in patients taking multiple antihypertensive drugs or heart failure drugs is unknown.

References

1. U.S. Food and Drug Administration. Information for healthcare professionals: Selective serotonin reuptake inhibitors (SSRIs), selective serotonin-norepinephrine reuptake inhibitors (SNRIs), 5-hydroxytryptamine receptor agonists (triptans), July 19, 2006.

2. MedGenMed. 2007 Sep 5;9(3):48.

3. QJM. 2003 Sep;96(9):635-42.

4. Am J Psychiatry. 1991 Jun;148(6):705-13.

5. Cephalalgia. 1999 Sep;19(7):668-75.

6. Headache. 2012 Feb;52(2):198-203.

7. Headache. 2012 Feb;52(2):195-7.

8. Headache. 2010 Jun;50(6):1089-99.

9. J Clin Pharmacol. 2007 Oct;47(10):1303-10.

10. J Urol. 2004 Nov;172(5 Pt 1):1935-40.

11. J Sex Med. 2012 Jun;9(6):1624-33.

Dr. Paauw is professor of medicine in the division of general internal medicine at the University of Washington, Seattle, and he serves as third-year medical student clerkship director at the University of Washington. Contact Dr. Paauw at dpaauw@uw.edu.

A 72-year-old man with benign prostatic hypertrophy comes to clinic to discuss recent problems with erectile dysfunction. He has been treated with tamsulosin with good results for the past 3 years for his BPH. He is given a prescription for vardenafil 10 mg for his ED. The pharmacist calls and asks if you want the prescription filled despite a drug interaction. What do you recommend?

A. Fill the prescription as written.

B. Have the patient take half a tablet of vardenafil.

C. Have the patient not take vardenafil within 6 hours of taking tamsulosin.

A 22-year-old woman presents with a unilateral headache, pounding in nature, worse with exercise. She is diagnosed with migraine. She has a history of depression and is taking 40 mg of fluoxetine. She is given a prescription for sumatriptan 100 mg. The pharmacist calls you and asks if you want to make changes because of possible drug interaction. What do you recommend?

A. Fill the prescription as written.

B. Have the patient take 50 mg of sumatriptan.

C. Have her reduce her fluoxetine dose to 20 mg.

D. Do not take sumatriptan within 12 hours of taking fluoxetine.

The title of this article is drug interaction myths. These are not true myths, but in both these cases, I think the prescriptions should be filled as written, and it will be safe for the patient to take the medications despite a theoretical drug interaction.

I have received calls from the pharmacist multiple times when I have prescribed these drug combinations, and I will share with you the evidence of safety for using these medications despite potential interactions.

In 2006, the Food and Drug Administration released an alert on serotonin syndrome occurring with combined use of selective serotonin reuptake inhibitors (SSRIs) or serotonin-norepinephrine reuptake inhibitors (SNRIs) with triptans.1 This alert was based on 29 cases that the FDA evaluated and felt justified an alert.

Dr. Randolph W. Evans did an analysis of all 29 cases to see if they met criteria for serotonin syndrome.2 He classified if the cases met two different criteria for serotonin syndrome: the Hunter criteria3 or the Sternbach criteria4.

Of the 29 case reports, 10 met the Sternbach criteria, and none of the reports met the Hunter criteria. Some of the cases included polypharmacy of other drugs that can cause serotonin syndrome. Two cases that met the Sternbach criteria were excluded because they were either not on an SSRI or had alternative compelling diagnoses.

Dr. Evans suggested the biologic implausibility of triptans causing serotonin syndrome, because serotonin syndrome is believed to be caused by activation of 5-HT1A and 5-HT2A receptors, whereas triptans act at the 5-HT1B/5-HT1D and 5-HT1F receptors.

In a prospective study of 12,339 patients with migraine who used subcutaneous sumatriptan for at least 1 year, 1,784 patients also received an SSRI.5 No episodes of serotonin syndrome were reported. David A. Sclar, Ph.D., and his colleagues estimated that in 2007-2008, 1.4 million patients were prescribed both a triptan and an SSRI or SNRI.6 That is a 36% increase from 2003-2004, despite a 50% reduction in coprescriptions from primary care physicians – suggesting neurologists were not affected by the FDA alert.7

The American Headache Society position paper on the FDA alert states, “The currently available evidence does not support limiting the use of triptans with SNRIs or SSRIs, or the use of triptan monotherapy, due to concerns for serotonin syndrome.”8

A warning will pop up on prescribing software when you prescribe a phosphodiesterase inhibitor in patients who are taking alpha-blockers. This is a common situation, because BPH and ED both become more common with age. The concern is that the combination of alpha-blocker plus phosphodiesterase inhibitor will increase the risk of hypotension.

Dr. Michel Guillaume and his colleagues studied the hemodynamic effect of doxazosin and tamsulosin in combination with tadalafil.9 A total of 45 healthy men aged 40-70 years were randomized to receive tadalafil and placebo for 28 days. Doxazosin was added after 7 days and continued for an additional 21 days. The second study included 39 men who received tadalafil and placebo for 7 days before adding tamsulosin for an additional 7 days.

There were no significant differences in change in standing systolic blood pressure with tadalafil with placebo, doxazosin, or tamsulosin.

Robert A. Kloner, M.D., Ph.D., and his colleagues reported on a randomized, double-blind, crossover trial of doxazosin 8 mg or placebo with tadalafil 20 mg and tamsulosin 0.4 mg or placebo with 10 mg or 20 mg of tadalafil.10 Tadalafil did augment the hypotensive effect of doxazosin, but it did not have any blood pressure effect on patients taking tamsulosin.

 

 

In a study of men taking both tamsulosin and vardenafil or tamsulosin and placebo for the treatment of BPH symptoms, Dr. Mauro Gacci and his colleagues found no significant difference in adverse effects in patients who received tamsulosin plus placebo, compared with men who received tamsulosin plus vardenafil.11

I think it is safe to prescribe triptans in patients who are on SSRIs and SNRIs. In patients who need both alpha-blockers and phosphodiesterase inhibitors, I think tamsulosin is the safest alpha-blocker option. It is best to not start a phosphodiesterase inhibitor at the same time as an alpha-blocker. The studies on coadministration of alpha-blockers and phosphodiesterase inhibitors have been done in either healthy volunteers, or in patients without severe systemic disease. So, the effect on blood pressure in patients taking multiple antihypertensive drugs or heart failure drugs is unknown.

References

1. U.S. Food and Drug Administration. Information for healthcare professionals: Selective serotonin reuptake inhibitors (SSRIs), selective serotonin-norepinephrine reuptake inhibitors (SNRIs), 5-hydroxytryptamine receptor agonists (triptans), July 19, 2006.

2. MedGenMed. 2007 Sep 5;9(3):48.

3. QJM. 2003 Sep;96(9):635-42.

4. Am J Psychiatry. 1991 Jun;148(6):705-13.

5. Cephalalgia. 1999 Sep;19(7):668-75.

6. Headache. 2012 Feb;52(2):198-203.

7. Headache. 2012 Feb;52(2):195-7.

8. Headache. 2010 Jun;50(6):1089-99.

9. J Clin Pharmacol. 2007 Oct;47(10):1303-10.

10. J Urol. 2004 Nov;172(5 Pt 1):1935-40.

11. J Sex Med. 2012 Jun;9(6):1624-33.

Dr. Paauw is professor of medicine in the division of general internal medicine at the University of Washington, Seattle, and he serves as third-year medical student clerkship director at the University of Washington. Contact Dr. Paauw at dpaauw@uw.edu.

Publications
Publications
Topics
Article Type
Display Headline
Drug interaction myths
Display Headline
Drug interaction myths
Legacy Keywords
durg interactions, SSRI, SNRI, antidepressant, migraine, triptan, sumatriptan, PDE5 inhibitor, hypertension, BPH, alpha blocker, Levitra, Cialis, Viagra
Legacy Keywords
durg interactions, SSRI, SNRI, antidepressant, migraine, triptan, sumatriptan, PDE5 inhibitor, hypertension, BPH, alpha blocker, Levitra, Cialis, Viagra
Sections
Disallow All Ads

Myth of the Month: Beta-blocker myths

Article Type
Changed
Fri, 01/18/2019 - 15:34
Display Headline
Myth of the Month: Beta-blocker myths

A 59-year-old man is admitted to the ICU with a myocardial infarction. He is discharged after 5 days on enalapril, metoprolol, simvastatin, and aspirin. At a 3-month follow-up, he is noted to have marked anhedonia, complaints of insomnia, feelings of worthlessness, and psychomotor retardation.

What would you do?

A) Stop the enalapril.

B) Stop the metoprolol.

C) Stop the simvastatin.

D) Begin a tricyclic antidepressant.

E) Begin an SSRI.

 

Dr. Douglas S. Paauw
Dr. Douglas S. Paauw

When I was in medical school, the dogma was to never give beta-blockers to patients with systolic heart failure, because it would worsen the heart failure.1 As we all know, this dogma completely reversed, and beta-blocker therapy is a cornerstone of treatment of patients with systolic heart failure, with improvements in morbidity and mortality.2 Underutilization of beta-blockers for indicated conditions is likely due to fear of beta-blocker side effects.2

There has long been concern that beta-blockers can cause, or worsen, depression. As a result, beta-blockers are sometimes withheld from patients with a history of depression who may benefit, or beta-blockers are stopped in patients who develop depression.

Early reports of possible beta-blocker–induced depression surfaced soon after the beta-blocker propranolol became available in the 1960s. A frequently cited reference is a letter to the British Medical Journal in which H.J. Waal reported that 20 of 89 patients on propranolol volunteered or exhibited depressive symptoms.3 Almost half of those patients were diagnosed with grade I depression – symptoms of irritability, insomnia, nightmares, and fatigue. No control group of patients was evaluated to ascertain the prevalence of those symptoms in patients treated with other antihypertensives, or in nonhypertensive patients.

M. H. Pollack and colleagues reported on a series of three patients who developed symptoms of depression after starting propranolol, and the researchers concluded that depression following the administration of propranolol was a real phenomenon.4

Many subsequent studies have cast doubt on the association of beta-blockers and depression. Depression is common following myocardial infarction and in patients with coronary artery disease. Several studies have looked closely for association with beta-blocker use in this population.

Dr. Steven J. Schleifer and colleagues evaluated 190 patients who had sustained a myocardial infarction for evidence of depression. The patients were interviewed 8-10 days after the infarct and again at 3 months. No antianginal or antihypertensive medications, including beta-blockers, were associated with an increase in depression.5

Dr. Joost P. van Melle and colleagues participated in a multicenter study that looked at patients following myocardial infarction, assessing for depressive symptoms at baseline and at 3, 6, 9, and 12 months using the Beck depression inventory.6 A total of 254 patients receiving beta-blockers were matched with 127 control patients post MI not receiving beta-blockers. No significant differences were found between non–beta-blocker users and beta-blocker users on the presence of depressive symptoms.

Robert Carney, Ph.D., and colleagues evaluated 75 patients undergoing elective cardiac catheterization with psychiatric interview and psychological assessments.7 Half of the patients in the study were receiving beta-blockers. Thirty-three percent of the patients who were not receiving beta-blockers met DSM-III criteria for depression, and 21% of the beta-blocker–treated patients met criteria for depression.

Dr. Linda Battes and colleagues reported that beta-blocker use actually decreased the risk of depression in patients who had undergone a percutaneous intervention, with a risk reduction of 49% for depression in beta-blocker–treated patients.8 In a study of elderly patients, Dr. Hendrika Luijendijk and colleagues followed 5,104 elderly persons for episodes of incident depression. They found that beta-blocker use did not increase the risk of developing depression.9

Beta-blockers often have been avoided in patients with obstructive pulmonary disease – both in patients with asthma and those with COPD – because of concern for worsening obstructive pulmonary disease. There is strong evidence now that beta-blocker use is not problematic in patients with COPD.

Dr. Surya Bhatt and colleagues found that beta-blocker use decreased COPD exacerbations.10 Almost 3,500 patients were included. During a median of 2.1 years of follow-up, beta-blocker use was associated with a significantly lower rate of total exacerbations (incidence risk ratio, 0.73; 95% confidence interval, 0.60-0.90; P = .003) and severe exacerbations (IRR, 0.67, 95% CI, 0.48-0.93; P = .016).

Dr. Qingxia Du and colleagues found that beta-blocker use in patients with COPD both reduced exacerbations and reduced mortality.11 In another study, the use of beta-blockers in patients hospitalized for acute exacerbations of COPD reduced mortality.12 Most of the patients receiving beta-blockers in that study had severe cardiovascular disease.

There are far fewer data on beta-blocker use in patients with asthma. In general, beta-blockers are routinely avoided in patients with asthma. In one small study of asthmatic patients receiving propranolol, there was no effect on methacholine challenge response, histamine responsiveness, or asthma control questionnaire results.13 In a murine model of asthma, long-term administration of beta-blockers resulted in a decrease in airway hyperresponsiveness, suggesting an anti-inflammatory effect.14 This topic is an area of interest for further study in asthma control.

 

 

So much of what we thought we knew about beta-blockers has turned out to not be so. We keep our eyes open and welcome further enlightenment.

References

1. Circulation. 1983 Jun;67(6 Pt 2):I91.

2. Expert Opin Drug Saf. 2015 Dec;14(12):1855-63.

3. Br Med J. 1967 Apr 1;2(5543):50.

4. J Nerv Ment Dis. 1985 Feb;173(2):118-9.

5. Am Heart J. 1991 May;121(5):1397-402.

6. J Am Coll Cardiol. 2006 Dec 5;48(11):2209-14.

7. Am J Med. 1987 Aug;83(2):223-6.

8. J Affect Disord. 2012 Feb;136(3):751-7.

9. J Clin Psychopharmacol. 2011 Feb;31(1):45-50.

10. Thorax. 2016 Jan;71(1):8-14.

11. PLoS ONE 9(11): e113048.

12. Thorax. 2008 Apr;63(4):301-5.

13. Am J Respir Crit Care Med. 2013 Jun 15;187(12):1308-14.

14. Int J Gen Med. 2013 Jul 8;6:549-55.

Dr. Paauw is professor of medicine in the division of general internal medicine at the University of Washington, Seattle, and he serves as third-year medical student clerkship director at the University of Washington. Contact Dr. Paauw at dpaauw@uw.edu.

Publications
Topics
Legacy Keywords
beta-blocker, heart attack, acute myocardial infarction, heart failure, COPD, asthma, propranolol, metoprolol
Sections

A 59-year-old man is admitted to the ICU with a myocardial infarction. He is discharged after 5 days on enalapril, metoprolol, simvastatin, and aspirin. At a 3-month follow-up, he is noted to have marked anhedonia, complaints of insomnia, feelings of worthlessness, and psychomotor retardation.

What would you do?

A) Stop the enalapril.

B) Stop the metoprolol.

C) Stop the simvastatin.

D) Begin a tricyclic antidepressant.

E) Begin an SSRI.

 

Dr. Douglas S. Paauw
Dr. Douglas S. Paauw

When I was in medical school, the dogma was to never give beta-blockers to patients with systolic heart failure, because it would worsen the heart failure.1 As we all know, this dogma completely reversed, and beta-blocker therapy is a cornerstone of treatment of patients with systolic heart failure, with improvements in morbidity and mortality.2 Underutilization of beta-blockers for indicated conditions is likely due to fear of beta-blocker side effects.2

There has long been concern that beta-blockers can cause, or worsen, depression. As a result, beta-blockers are sometimes withheld from patients with a history of depression who may benefit, or beta-blockers are stopped in patients who develop depression.

Early reports of possible beta-blocker–induced depression surfaced soon after the beta-blocker propranolol became available in the 1960s. A frequently cited reference is a letter to the British Medical Journal in which H.J. Waal reported that 20 of 89 patients on propranolol volunteered or exhibited depressive symptoms.3 Almost half of those patients were diagnosed with grade I depression – symptoms of irritability, insomnia, nightmares, and fatigue. No control group of patients was evaluated to ascertain the prevalence of those symptoms in patients treated with other antihypertensives, or in nonhypertensive patients.

M. H. Pollack and colleagues reported on a series of three patients who developed symptoms of depression after starting propranolol, and the researchers concluded that depression following the administration of propranolol was a real phenomenon.4

Many subsequent studies have cast doubt on the association of beta-blockers and depression. Depression is common following myocardial infarction and in patients with coronary artery disease. Several studies have looked closely for association with beta-blocker use in this population.

Dr. Steven J. Schleifer and colleagues evaluated 190 patients who had sustained a myocardial infarction for evidence of depression. The patients were interviewed 8-10 days after the infarct and again at 3 months. No antianginal or antihypertensive medications, including beta-blockers, were associated with an increase in depression.5

Dr. Joost P. van Melle and colleagues participated in a multicenter study that looked at patients following myocardial infarction, assessing for depressive symptoms at baseline and at 3, 6, 9, and 12 months using the Beck depression inventory.6 A total of 254 patients receiving beta-blockers were matched with 127 control patients post MI not receiving beta-blockers. No significant differences were found between non–beta-blocker users and beta-blocker users on the presence of depressive symptoms.

Robert Carney, Ph.D., and colleagues evaluated 75 patients undergoing elective cardiac catheterization with psychiatric interview and psychological assessments.7 Half of the patients in the study were receiving beta-blockers. Thirty-three percent of the patients who were not receiving beta-blockers met DSM-III criteria for depression, and 21% of the beta-blocker–treated patients met criteria for depression.

Dr. Linda Battes and colleagues reported that beta-blocker use actually decreased the risk of depression in patients who had undergone a percutaneous intervention, with a risk reduction of 49% for depression in beta-blocker–treated patients.8 In a study of elderly patients, Dr. Hendrika Luijendijk and colleagues followed 5,104 elderly persons for episodes of incident depression. They found that beta-blocker use did not increase the risk of developing depression.9

Beta-blockers often have been avoided in patients with obstructive pulmonary disease – both in patients with asthma and those with COPD – because of concern for worsening obstructive pulmonary disease. There is strong evidence now that beta-blocker use is not problematic in patients with COPD.

Dr. Surya Bhatt and colleagues found that beta-blocker use decreased COPD exacerbations.10 Almost 3,500 patients were included. During a median of 2.1 years of follow-up, beta-blocker use was associated with a significantly lower rate of total exacerbations (incidence risk ratio, 0.73; 95% confidence interval, 0.60-0.90; P = .003) and severe exacerbations (IRR, 0.67, 95% CI, 0.48-0.93; P = .016).

Dr. Qingxia Du and colleagues found that beta-blocker use in patients with COPD both reduced exacerbations and reduced mortality.11 In another study, the use of beta-blockers in patients hospitalized for acute exacerbations of COPD reduced mortality.12 Most of the patients receiving beta-blockers in that study had severe cardiovascular disease.

There are far fewer data on beta-blocker use in patients with asthma. In general, beta-blockers are routinely avoided in patients with asthma. In one small study of asthmatic patients receiving propranolol, there was no effect on methacholine challenge response, histamine responsiveness, or asthma control questionnaire results.13 In a murine model of asthma, long-term administration of beta-blockers resulted in a decrease in airway hyperresponsiveness, suggesting an anti-inflammatory effect.14 This topic is an area of interest for further study in asthma control.

 

 

So much of what we thought we knew about beta-blockers has turned out to not be so. We keep our eyes open and welcome further enlightenment.

References

1. Circulation. 1983 Jun;67(6 Pt 2):I91.

2. Expert Opin Drug Saf. 2015 Dec;14(12):1855-63.

3. Br Med J. 1967 Apr 1;2(5543):50.

4. J Nerv Ment Dis. 1985 Feb;173(2):118-9.

5. Am Heart J. 1991 May;121(5):1397-402.

6. J Am Coll Cardiol. 2006 Dec 5;48(11):2209-14.

7. Am J Med. 1987 Aug;83(2):223-6.

8. J Affect Disord. 2012 Feb;136(3):751-7.

9. J Clin Psychopharmacol. 2011 Feb;31(1):45-50.

10. Thorax. 2016 Jan;71(1):8-14.

11. PLoS ONE 9(11): e113048.

12. Thorax. 2008 Apr;63(4):301-5.

13. Am J Respir Crit Care Med. 2013 Jun 15;187(12):1308-14.

14. Int J Gen Med. 2013 Jul 8;6:549-55.

Dr. Paauw is professor of medicine in the division of general internal medicine at the University of Washington, Seattle, and he serves as third-year medical student clerkship director at the University of Washington. Contact Dr. Paauw at dpaauw@uw.edu.

A 59-year-old man is admitted to the ICU with a myocardial infarction. He is discharged after 5 days on enalapril, metoprolol, simvastatin, and aspirin. At a 3-month follow-up, he is noted to have marked anhedonia, complaints of insomnia, feelings of worthlessness, and psychomotor retardation.

What would you do?

A) Stop the enalapril.

B) Stop the metoprolol.

C) Stop the simvastatin.

D) Begin a tricyclic antidepressant.

E) Begin an SSRI.

 

Dr. Douglas S. Paauw
Dr. Douglas S. Paauw

When I was in medical school, the dogma was to never give beta-blockers to patients with systolic heart failure, because it would worsen the heart failure.1 As we all know, this dogma completely reversed, and beta-blocker therapy is a cornerstone of treatment of patients with systolic heart failure, with improvements in morbidity and mortality.2 Underutilization of beta-blockers for indicated conditions is likely due to fear of beta-blocker side effects.2

There has long been concern that beta-blockers can cause, or worsen, depression. As a result, beta-blockers are sometimes withheld from patients with a history of depression who may benefit, or beta-blockers are stopped in patients who develop depression.

Early reports of possible beta-blocker–induced depression surfaced soon after the beta-blocker propranolol became available in the 1960s. A frequently cited reference is a letter to the British Medical Journal in which H.J. Waal reported that 20 of 89 patients on propranolol volunteered or exhibited depressive symptoms.3 Almost half of those patients were diagnosed with grade I depression – symptoms of irritability, insomnia, nightmares, and fatigue. No control group of patients was evaluated to ascertain the prevalence of those symptoms in patients treated with other antihypertensives, or in nonhypertensive patients.

M. H. Pollack and colleagues reported on a series of three patients who developed symptoms of depression after starting propranolol, and the researchers concluded that depression following the administration of propranolol was a real phenomenon.4

Many subsequent studies have cast doubt on the association of beta-blockers and depression. Depression is common following myocardial infarction and in patients with coronary artery disease. Several studies have looked closely for association with beta-blocker use in this population.

Dr. Steven J. Schleifer and colleagues evaluated 190 patients who had sustained a myocardial infarction for evidence of depression. The patients were interviewed 8-10 days after the infarct and again at 3 months. No antianginal or antihypertensive medications, including beta-blockers, were associated with an increase in depression.5

Dr. Joost P. van Melle and colleagues participated in a multicenter study that looked at patients following myocardial infarction, assessing for depressive symptoms at baseline and at 3, 6, 9, and 12 months using the Beck depression inventory.6 A total of 254 patients receiving beta-blockers were matched with 127 control patients post MI not receiving beta-blockers. No significant differences were found between non–beta-blocker users and beta-blocker users on the presence of depressive symptoms.

Robert Carney, Ph.D., and colleagues evaluated 75 patients undergoing elective cardiac catheterization with psychiatric interview and psychological assessments.7 Half of the patients in the study were receiving beta-blockers. Thirty-three percent of the patients who were not receiving beta-blockers met DSM-III criteria for depression, and 21% of the beta-blocker–treated patients met criteria for depression.

Dr. Linda Battes and colleagues reported that beta-blocker use actually decreased the risk of depression in patients who had undergone a percutaneous intervention, with a risk reduction of 49% for depression in beta-blocker–treated patients.8 In a study of elderly patients, Dr. Hendrika Luijendijk and colleagues followed 5,104 elderly persons for episodes of incident depression. They found that beta-blocker use did not increase the risk of developing depression.9

Beta-blockers often have been avoided in patients with obstructive pulmonary disease – both in patients with asthma and those with COPD – because of concern for worsening obstructive pulmonary disease. There is strong evidence now that beta-blocker use is not problematic in patients with COPD.

Dr. Surya Bhatt and colleagues found that beta-blocker use decreased COPD exacerbations.10 Almost 3,500 patients were included. During a median of 2.1 years of follow-up, beta-blocker use was associated with a significantly lower rate of total exacerbations (incidence risk ratio, 0.73; 95% confidence interval, 0.60-0.90; P = .003) and severe exacerbations (IRR, 0.67, 95% CI, 0.48-0.93; P = .016).

Dr. Qingxia Du and colleagues found that beta-blocker use in patients with COPD both reduced exacerbations and reduced mortality.11 In another study, the use of beta-blockers in patients hospitalized for acute exacerbations of COPD reduced mortality.12 Most of the patients receiving beta-blockers in that study had severe cardiovascular disease.

There are far fewer data on beta-blocker use in patients with asthma. In general, beta-blockers are routinely avoided in patients with asthma. In one small study of asthmatic patients receiving propranolol, there was no effect on methacholine challenge response, histamine responsiveness, or asthma control questionnaire results.13 In a murine model of asthma, long-term administration of beta-blockers resulted in a decrease in airway hyperresponsiveness, suggesting an anti-inflammatory effect.14 This topic is an area of interest for further study in asthma control.

 

 

So much of what we thought we knew about beta-blockers has turned out to not be so. We keep our eyes open and welcome further enlightenment.

References

1. Circulation. 1983 Jun;67(6 Pt 2):I91.

2. Expert Opin Drug Saf. 2015 Dec;14(12):1855-63.

3. Br Med J. 1967 Apr 1;2(5543):50.

4. J Nerv Ment Dis. 1985 Feb;173(2):118-9.

5. Am Heart J. 1991 May;121(5):1397-402.

6. J Am Coll Cardiol. 2006 Dec 5;48(11):2209-14.

7. Am J Med. 1987 Aug;83(2):223-6.

8. J Affect Disord. 2012 Feb;136(3):751-7.

9. J Clin Psychopharmacol. 2011 Feb;31(1):45-50.

10. Thorax. 2016 Jan;71(1):8-14.

11. PLoS ONE 9(11): e113048.

12. Thorax. 2008 Apr;63(4):301-5.

13. Am J Respir Crit Care Med. 2013 Jun 15;187(12):1308-14.

14. Int J Gen Med. 2013 Jul 8;6:549-55.

Dr. Paauw is professor of medicine in the division of general internal medicine at the University of Washington, Seattle, and he serves as third-year medical student clerkship director at the University of Washington. Contact Dr. Paauw at dpaauw@uw.edu.

Publications
Publications
Topics
Article Type
Display Headline
Myth of the Month: Beta-blocker myths
Display Headline
Myth of the Month: Beta-blocker myths
Legacy Keywords
beta-blocker, heart attack, acute myocardial infarction, heart failure, COPD, asthma, propranolol, metoprolol
Legacy Keywords
beta-blocker, heart attack, acute myocardial infarction, heart failure, COPD, asthma, propranolol, metoprolol
Sections
Disallow All Ads

Myth of the Month: Does Colace work?

Article Type
Changed
Wed, 05/26/2021 - 13:55
Display Headline
Myth of the Month: Does Colace work?

Myth: Docusate is a stool softener and helps with constipation.

A 60-year-old man is injured in a fall and breaks four ribs. He is in severe pain and is prescribed oxycodone and naproxen for pain. What treatment would you prescribe to help decrease problems with constipation?

A. Docusate.

B. Docusate and polyethylene glycol.

C. Psyllium.

D. Polyethylene glycol.

Constipation is extremely common, occurring in up to 20%-25% of the elderly population and 90% of patients treated with opioids. The formal definition of constipation is fewer than three bowel movements per week. Patients are concerned with other symptoms as well, including hard stool consistency and the feeling of incomplete evacuation.

An extremely commonly prescribed medication for patients with symptoms of constipation/hard stool passage is docusate (Colace). This medication is often a part of bowel programs for institutionalized/hospitalized patients, as well as being frequently prescribed when patients are treated with opiates.

Does it work?

 

Dr. Douglas S. Paauw
Dr. Douglas S. Paauw

Docusate is frequently prescribed as a “stool softener,” but does it increase water content in stool? In a randomized, controlled trial of docusate vs. psyllium, 170 adult patients with chronic constipation received either 5.1 g twice a day of psyllium or 100 mg twice a day of docusate (Aliment Pharmacol Ther. 1998 May;12[5]:491-7).

Psyllium was superior in its effect on stool frequency, stool water content, total stool output, and the combination of several objective measures of constipation. Compared with baseline, psyllium increased stool water content by 2.33%, vs .01% for docusate (P =. 007), and stool weight was increased in the group treated with psyllium, compared with docusate-treated patients (359.9 g/week vs. 271.9 g/week, respectively; P = .005). Docusate does not appear to have any effect on stool water content or amount of stool.

In a study of constipation treatment in patients receiving opioids, Dr. Yoko Tarumi and her colleagues studied 74 patients admitted to hospice units (J Pain Symptom Manage. 2013 Jan;45[1]:2-13). A total of 74 patients were randomized to receive docusate 100 mg twice a day plus senna, or placebo plus senna. Once the study was started, inclusion criteria were broadened to include hospice patients with nonmalignant disease and patients who were not on opioids.

Almost all patients in the study did receive opioids (94% of the docusate patients and 100% of placebo-treated patients). There were no significant between the groups in stool volume, frequency, consistency, or in perceived completeness of evacuation.

In a randomized, controlled study of elderly patients on a medicine ward, 34 patients were randomized to docusate or control (no laxatives)(J Chronic Dis. 1976 Jan;29[1]:59-63). There was no difference in frequency or quality of stools between groups.

A systematic review of the usefulness of docusate in chronically ill patients concluded that the widespread use of docusate for the treatment of constipation in palliative-care patients is based on inadequate experimental evidence (J Pain Symptom Manage. 2000 Feb;19[2]:130-6).

The Canadian Agency for Drugs and Technologies in Health concluded “the available evidence suggests that docusate is no more effective than placebo in the prevention or management of constipation” (Dioctyl sulfosuccinate or docusate [calcium or sodium] for the prevention or management of constipation: a review of the clinical effectiveness. Canadian Agency for Drugs and Technologies in Health; 2014 Jun 26).

Dr. Davendra Ramkumar and his colleagues published a systematic review of drug trials for the treatment of constipation in 2005 (Am J Gastroenterol. 2005 Apr;100[4]:936-71). Only polyethylene glycol and tegaserod received grade A evidence for published trials. Psyllium and lactulose received grade B evidence. Docusate received a level 3, grade C for evidence (poor quality evidence, poor evidence to support a recommendation for or against the use of the modality).

I have been surprised at how docusate has been the most commonly prescribed laxative agent. Polyethylene glycol or psyllium are better evidence-based options. Docusate is often prescribed as a stool softener, and it has even less evidence that it softens stool than its poor evidence as a laxative.

Acknowledgments

My thanks to the late Dr. David Saunders for teaching me 30 years ago that docusate was not a helpful option for the management of constipation, and to Sarah Steinkruger for doing much of the research that was used in this column.

Dr. Paauw is professor of medicine in the division of general internal medicine at the University of Washington, Seattle, and he serves as third-year medical student clerkship director at the University of Washington. Contact Dr. Paauw at dpaauw@uw.edu.

Publications
Topics
Legacy Keywords
Colace, constipation, opioids, docusate, psyllium, stool softener
Sections

Myth: Docusate is a stool softener and helps with constipation.

A 60-year-old man is injured in a fall and breaks four ribs. He is in severe pain and is prescribed oxycodone and naproxen for pain. What treatment would you prescribe to help decrease problems with constipation?

A. Docusate.

B. Docusate and polyethylene glycol.

C. Psyllium.

D. Polyethylene glycol.

Constipation is extremely common, occurring in up to 20%-25% of the elderly population and 90% of patients treated with opioids. The formal definition of constipation is fewer than three bowel movements per week. Patients are concerned with other symptoms as well, including hard stool consistency and the feeling of incomplete evacuation.

An extremely commonly prescribed medication for patients with symptoms of constipation/hard stool passage is docusate (Colace). This medication is often a part of bowel programs for institutionalized/hospitalized patients, as well as being frequently prescribed when patients are treated with opiates.

Does it work?

 

Dr. Douglas S. Paauw
Dr. Douglas S. Paauw

Docusate is frequently prescribed as a “stool softener,” but does it increase water content in stool? In a randomized, controlled trial of docusate vs. psyllium, 170 adult patients with chronic constipation received either 5.1 g twice a day of psyllium or 100 mg twice a day of docusate (Aliment Pharmacol Ther. 1998 May;12[5]:491-7).

Psyllium was superior in its effect on stool frequency, stool water content, total stool output, and the combination of several objective measures of constipation. Compared with baseline, psyllium increased stool water content by 2.33%, vs .01% for docusate (P =. 007), and stool weight was increased in the group treated with psyllium, compared with docusate-treated patients (359.9 g/week vs. 271.9 g/week, respectively; P = .005). Docusate does not appear to have any effect on stool water content or amount of stool.

In a study of constipation treatment in patients receiving opioids, Dr. Yoko Tarumi and her colleagues studied 74 patients admitted to hospice units (J Pain Symptom Manage. 2013 Jan;45[1]:2-13). A total of 74 patients were randomized to receive docusate 100 mg twice a day plus senna, or placebo plus senna. Once the study was started, inclusion criteria were broadened to include hospice patients with nonmalignant disease and patients who were not on opioids.

Almost all patients in the study did receive opioids (94% of the docusate patients and 100% of placebo-treated patients). There were no significant between the groups in stool volume, frequency, consistency, or in perceived completeness of evacuation.

In a randomized, controlled study of elderly patients on a medicine ward, 34 patients were randomized to docusate or control (no laxatives)(J Chronic Dis. 1976 Jan;29[1]:59-63). There was no difference in frequency or quality of stools between groups.

A systematic review of the usefulness of docusate in chronically ill patients concluded that the widespread use of docusate for the treatment of constipation in palliative-care patients is based on inadequate experimental evidence (J Pain Symptom Manage. 2000 Feb;19[2]:130-6).

The Canadian Agency for Drugs and Technologies in Health concluded “the available evidence suggests that docusate is no more effective than placebo in the prevention or management of constipation” (Dioctyl sulfosuccinate or docusate [calcium or sodium] for the prevention or management of constipation: a review of the clinical effectiveness. Canadian Agency for Drugs and Technologies in Health; 2014 Jun 26).

Dr. Davendra Ramkumar and his colleagues published a systematic review of drug trials for the treatment of constipation in 2005 (Am J Gastroenterol. 2005 Apr;100[4]:936-71). Only polyethylene glycol and tegaserod received grade A evidence for published trials. Psyllium and lactulose received grade B evidence. Docusate received a level 3, grade C for evidence (poor quality evidence, poor evidence to support a recommendation for or against the use of the modality).

I have been surprised at how docusate has been the most commonly prescribed laxative agent. Polyethylene glycol or psyllium are better evidence-based options. Docusate is often prescribed as a stool softener, and it has even less evidence that it softens stool than its poor evidence as a laxative.

Acknowledgments

My thanks to the late Dr. David Saunders for teaching me 30 years ago that docusate was not a helpful option for the management of constipation, and to Sarah Steinkruger for doing much of the research that was used in this column.

Dr. Paauw is professor of medicine in the division of general internal medicine at the University of Washington, Seattle, and he serves as third-year medical student clerkship director at the University of Washington. Contact Dr. Paauw at dpaauw@uw.edu.

Myth: Docusate is a stool softener and helps with constipation.

A 60-year-old man is injured in a fall and breaks four ribs. He is in severe pain and is prescribed oxycodone and naproxen for pain. What treatment would you prescribe to help decrease problems with constipation?

A. Docusate.

B. Docusate and polyethylene glycol.

C. Psyllium.

D. Polyethylene glycol.

Constipation is extremely common, occurring in up to 20%-25% of the elderly population and 90% of patients treated with opioids. The formal definition of constipation is fewer than three bowel movements per week. Patients are concerned with other symptoms as well, including hard stool consistency and the feeling of incomplete evacuation.

An extremely commonly prescribed medication for patients with symptoms of constipation/hard stool passage is docusate (Colace). This medication is often a part of bowel programs for institutionalized/hospitalized patients, as well as being frequently prescribed when patients are treated with opiates.

Does it work?

 

Dr. Douglas S. Paauw
Dr. Douglas S. Paauw

Docusate is frequently prescribed as a “stool softener,” but does it increase water content in stool? In a randomized, controlled trial of docusate vs. psyllium, 170 adult patients with chronic constipation received either 5.1 g twice a day of psyllium or 100 mg twice a day of docusate (Aliment Pharmacol Ther. 1998 May;12[5]:491-7).

Psyllium was superior in its effect on stool frequency, stool water content, total stool output, and the combination of several objective measures of constipation. Compared with baseline, psyllium increased stool water content by 2.33%, vs .01% for docusate (P =. 007), and stool weight was increased in the group treated with psyllium, compared with docusate-treated patients (359.9 g/week vs. 271.9 g/week, respectively; P = .005). Docusate does not appear to have any effect on stool water content or amount of stool.

In a study of constipation treatment in patients receiving opioids, Dr. Yoko Tarumi and her colleagues studied 74 patients admitted to hospice units (J Pain Symptom Manage. 2013 Jan;45[1]:2-13). A total of 74 patients were randomized to receive docusate 100 mg twice a day plus senna, or placebo plus senna. Once the study was started, inclusion criteria were broadened to include hospice patients with nonmalignant disease and patients who were not on opioids.

Almost all patients in the study did receive opioids (94% of the docusate patients and 100% of placebo-treated patients). There were no significant between the groups in stool volume, frequency, consistency, or in perceived completeness of evacuation.

In a randomized, controlled study of elderly patients on a medicine ward, 34 patients were randomized to docusate or control (no laxatives)(J Chronic Dis. 1976 Jan;29[1]:59-63). There was no difference in frequency or quality of stools between groups.

A systematic review of the usefulness of docusate in chronically ill patients concluded that the widespread use of docusate for the treatment of constipation in palliative-care patients is based on inadequate experimental evidence (J Pain Symptom Manage. 2000 Feb;19[2]:130-6).

The Canadian Agency for Drugs and Technologies in Health concluded “the available evidence suggests that docusate is no more effective than placebo in the prevention or management of constipation” (Dioctyl sulfosuccinate or docusate [calcium or sodium] for the prevention or management of constipation: a review of the clinical effectiveness. Canadian Agency for Drugs and Technologies in Health; 2014 Jun 26).

Dr. Davendra Ramkumar and his colleagues published a systematic review of drug trials for the treatment of constipation in 2005 (Am J Gastroenterol. 2005 Apr;100[4]:936-71). Only polyethylene glycol and tegaserod received grade A evidence for published trials. Psyllium and lactulose received grade B evidence. Docusate received a level 3, grade C for evidence (poor quality evidence, poor evidence to support a recommendation for or against the use of the modality).

I have been surprised at how docusate has been the most commonly prescribed laxative agent. Polyethylene glycol or psyllium are better evidence-based options. Docusate is often prescribed as a stool softener, and it has even less evidence that it softens stool than its poor evidence as a laxative.

Acknowledgments

My thanks to the late Dr. David Saunders for teaching me 30 years ago that docusate was not a helpful option for the management of constipation, and to Sarah Steinkruger for doing much of the research that was used in this column.

Dr. Paauw is professor of medicine in the division of general internal medicine at the University of Washington, Seattle, and he serves as third-year medical student clerkship director at the University of Washington. Contact Dr. Paauw at dpaauw@uw.edu.

Publications
Publications
Topics
Article Type
Display Headline
Myth of the Month: Does Colace work?
Display Headline
Myth of the Month: Does Colace work?
Legacy Keywords
Colace, constipation, opioids, docusate, psyllium, stool softener
Legacy Keywords
Colace, constipation, opioids, docusate, psyllium, stool softener
Sections
Disallow All Ads

Myth of the Month: Why can’t I give my patient hydrochlorothiazide?

Article Type
Changed
Fri, 01/18/2019 - 15:01
Display Headline
Myth of the Month: Why can’t I give my patient hydrochlorothiazide?

A 56-year-old woman presents with hypertension. Her blood pressures have been about 160/100 mm Hg. Past medical history includes depression, GERD, and hypothyroidism, and the patient has an allergy to sulfamethoxazole (rash).

She receives a prescription for hydrochlorothiazide. The pharmacist calls later to report that the pharmacy did not fill the prescription because the patient has a sulfa allergy.

What would you recommend?

A. Call in a prescription for a calcium channel blocker.

B. Call in a prescription for an ACE inhibitor.

C. Call in a prescription for a beta-blocker.

D. Call and ask the pharmacist to fill the hydrochlorothiazide prescription.

I have received this call from pharmacies many times. The patients are usually very frustrated because they could not pick up their medications. Is this the right call from the pharmacist? Does the fact that the patient has a sulfa allergy make prescribing hydrochlorothiazide wrong?

Allergies to sulfonamide antibiotics occur in about 3% of patients who are prescribed the drugs. Sulfa-containing antibiotics contain a five- or six-member nitrogen-containing ring attached to the N1 nitrogen of the sulfonamide group and an arylamine group (H2N) at the N4 position of the sulfonamide group (Pharmacotherapy 2004;24:856-70). Sulfa-containing nonantibiotics – including thiazides and loop diuretics, as well as COX-2 inhibitors – do not contain these same features.

There has always been concern that there is a possibility of increased risk of drug reactions in patients who receive a sulfa nonantibiotic, and frequently prescriptions for these medicines are not filled by pharmacies without directly confirming the intent by speaking with the prescriber.

Brian Hemstreet, Pharm.D., and Robert Page II, Pharm.D., did a prospective, observational study of patients admitted to a hospital with a history of sulfa allergy (Pharmacotherapy 2006;26:551-7). A total of 94 patients were studied who had a reported sulfa allergy. Forty of these patients were taking a sulfonamide nonantibiotic at the time of admission (most commonly furosemide). Nine of the patients received a sulfonamide nonantibiotic during their hospitalization. None of the patients had a drug reaction, either while receiving a sulfonamide nonantibiotic in the hospital, or previously while receiving one as an outpatient.

Dr. Pilar Tornero and colleagues used patch testing and control oral challenge in patients with previous fixed drug eruptions to trimethoprim-sulfamethoxazole or an unknown sulfonamide (Contact Dermatitis 2004;51:57-62). All patients received low doses of oral sulfonamide antibiotics (trimethoprim-sulfamethoxazole, sulfadiazine, or sulfamethizole). All patients also received furosemide.

Every patient with previous known sulfa reaction had a positive oral challenge test when given sulfamethoxazole. There was cross reactivity with other sulfa antibiotics: Seven of 18 patients with prior sulfamethoxazole allergy reacted to oral challenge with sulfadiazine, and 4 of 9 patients with prior allergy with sulfamethoxazole reacted to challenge with sulfamethazine. All 28 patients in the study were challenged with furosemide (a sulfa nonantibiotic) with no allergic reactions.

Dr. Brian Strom and colleagues conducted a retrospective cohort study using general practice research database, looking at risk of allergic reactions within 30 days of receipt of a sulfonamide nonantibiotic (N. Engl. J. Med. 2003;349:1628-35).

Patients who had a history of prior hypersensitivity to a sulfonamide antibiotic were compared with those with no previous history of allergy. Analyses were also performed in patients with a prior penicillin allergy. A total of 969 patients who had an allergic reaction after a sulfonamide antibiotic were evaluated.

Of those patients, 9.1% had an allergic reaction after receiving a sulfonamide nonantibiotic. In those patients without a sulfa antibiotic allergy, only 1.6% had a reaction to a sulfa nonantibiotic. Interestingly, in patients with a prior history of penicillin reaction, 14.6% had an allergic reaction when receiving a sulfa nonantibiotic.

Patients with a history of sulfa allergy to a sulfa antibiotic are more likely to have a reaction to a sulfa nonantibiotic than are those without a previous allergy history. But this appears to be due to overall increased reactiveness and not a cross reactivity, because those with history of penicillin allergy had an even higher allergy rate to sulfa nonantibiotics than did patients with a prior sulfa allergy.

 

Dr. Paauw is professor of medicine in the division of general internal medicine at the University of Washington, Seattle, and he serves as third-year medical student clerkship director at the University of Washington. Contact Dr. Paauw at dpaauw@uw.edu.

Publications
Topics
Legacy Keywords
sulfa drug, drug allergy, hydrochlorothazide, high blood pressure
Sections

A 56-year-old woman presents with hypertension. Her blood pressures have been about 160/100 mm Hg. Past medical history includes depression, GERD, and hypothyroidism, and the patient has an allergy to sulfamethoxazole (rash).

She receives a prescription for hydrochlorothiazide. The pharmacist calls later to report that the pharmacy did not fill the prescription because the patient has a sulfa allergy.

What would you recommend?

A. Call in a prescription for a calcium channel blocker.

B. Call in a prescription for an ACE inhibitor.

C. Call in a prescription for a beta-blocker.

D. Call and ask the pharmacist to fill the hydrochlorothiazide prescription.

I have received this call from pharmacies many times. The patients are usually very frustrated because they could not pick up their medications. Is this the right call from the pharmacist? Does the fact that the patient has a sulfa allergy make prescribing hydrochlorothiazide wrong?

Allergies to sulfonamide antibiotics occur in about 3% of patients who are prescribed the drugs. Sulfa-containing antibiotics contain a five- or six-member nitrogen-containing ring attached to the N1 nitrogen of the sulfonamide group and an arylamine group (H2N) at the N4 position of the sulfonamide group (Pharmacotherapy 2004;24:856-70). Sulfa-containing nonantibiotics – including thiazides and loop diuretics, as well as COX-2 inhibitors – do not contain these same features.

There has always been concern that there is a possibility of increased risk of drug reactions in patients who receive a sulfa nonantibiotic, and frequently prescriptions for these medicines are not filled by pharmacies without directly confirming the intent by speaking with the prescriber.

Brian Hemstreet, Pharm.D., and Robert Page II, Pharm.D., did a prospective, observational study of patients admitted to a hospital with a history of sulfa allergy (Pharmacotherapy 2006;26:551-7). A total of 94 patients were studied who had a reported sulfa allergy. Forty of these patients were taking a sulfonamide nonantibiotic at the time of admission (most commonly furosemide). Nine of the patients received a sulfonamide nonantibiotic during their hospitalization. None of the patients had a drug reaction, either while receiving a sulfonamide nonantibiotic in the hospital, or previously while receiving one as an outpatient.

Dr. Pilar Tornero and colleagues used patch testing and control oral challenge in patients with previous fixed drug eruptions to trimethoprim-sulfamethoxazole or an unknown sulfonamide (Contact Dermatitis 2004;51:57-62). All patients received low doses of oral sulfonamide antibiotics (trimethoprim-sulfamethoxazole, sulfadiazine, or sulfamethizole). All patients also received furosemide.

Every patient with previous known sulfa reaction had a positive oral challenge test when given sulfamethoxazole. There was cross reactivity with other sulfa antibiotics: Seven of 18 patients with prior sulfamethoxazole allergy reacted to oral challenge with sulfadiazine, and 4 of 9 patients with prior allergy with sulfamethoxazole reacted to challenge with sulfamethazine. All 28 patients in the study were challenged with furosemide (a sulfa nonantibiotic) with no allergic reactions.

Dr. Brian Strom and colleagues conducted a retrospective cohort study using general practice research database, looking at risk of allergic reactions within 30 days of receipt of a sulfonamide nonantibiotic (N. Engl. J. Med. 2003;349:1628-35).

Patients who had a history of prior hypersensitivity to a sulfonamide antibiotic were compared with those with no previous history of allergy. Analyses were also performed in patients with a prior penicillin allergy. A total of 969 patients who had an allergic reaction after a sulfonamide antibiotic were evaluated.

Of those patients, 9.1% had an allergic reaction after receiving a sulfonamide nonantibiotic. In those patients without a sulfa antibiotic allergy, only 1.6% had a reaction to a sulfa nonantibiotic. Interestingly, in patients with a prior history of penicillin reaction, 14.6% had an allergic reaction when receiving a sulfa nonantibiotic.

Patients with a history of sulfa allergy to a sulfa antibiotic are more likely to have a reaction to a sulfa nonantibiotic than are those without a previous allergy history. But this appears to be due to overall increased reactiveness and not a cross reactivity, because those with history of penicillin allergy had an even higher allergy rate to sulfa nonantibiotics than did patients with a prior sulfa allergy.

 

Dr. Paauw is professor of medicine in the division of general internal medicine at the University of Washington, Seattle, and he serves as third-year medical student clerkship director at the University of Washington. Contact Dr. Paauw at dpaauw@uw.edu.

A 56-year-old woman presents with hypertension. Her blood pressures have been about 160/100 mm Hg. Past medical history includes depression, GERD, and hypothyroidism, and the patient has an allergy to sulfamethoxazole (rash).

She receives a prescription for hydrochlorothiazide. The pharmacist calls later to report that the pharmacy did not fill the prescription because the patient has a sulfa allergy.

What would you recommend?

A. Call in a prescription for a calcium channel blocker.

B. Call in a prescription for an ACE inhibitor.

C. Call in a prescription for a beta-blocker.

D. Call and ask the pharmacist to fill the hydrochlorothiazide prescription.

I have received this call from pharmacies many times. The patients are usually very frustrated because they could not pick up their medications. Is this the right call from the pharmacist? Does the fact that the patient has a sulfa allergy make prescribing hydrochlorothiazide wrong?

Allergies to sulfonamide antibiotics occur in about 3% of patients who are prescribed the drugs. Sulfa-containing antibiotics contain a five- or six-member nitrogen-containing ring attached to the N1 nitrogen of the sulfonamide group and an arylamine group (H2N) at the N4 position of the sulfonamide group (Pharmacotherapy 2004;24:856-70). Sulfa-containing nonantibiotics – including thiazides and loop diuretics, as well as COX-2 inhibitors – do not contain these same features.

There has always been concern that there is a possibility of increased risk of drug reactions in patients who receive a sulfa nonantibiotic, and frequently prescriptions for these medicines are not filled by pharmacies without directly confirming the intent by speaking with the prescriber.

Brian Hemstreet, Pharm.D., and Robert Page II, Pharm.D., did a prospective, observational study of patients admitted to a hospital with a history of sulfa allergy (Pharmacotherapy 2006;26:551-7). A total of 94 patients were studied who had a reported sulfa allergy. Forty of these patients were taking a sulfonamide nonantibiotic at the time of admission (most commonly furosemide). Nine of the patients received a sulfonamide nonantibiotic during their hospitalization. None of the patients had a drug reaction, either while receiving a sulfonamide nonantibiotic in the hospital, or previously while receiving one as an outpatient.

Dr. Pilar Tornero and colleagues used patch testing and control oral challenge in patients with previous fixed drug eruptions to trimethoprim-sulfamethoxazole or an unknown sulfonamide (Contact Dermatitis 2004;51:57-62). All patients received low doses of oral sulfonamide antibiotics (trimethoprim-sulfamethoxazole, sulfadiazine, or sulfamethizole). All patients also received furosemide.

Every patient with previous known sulfa reaction had a positive oral challenge test when given sulfamethoxazole. There was cross reactivity with other sulfa antibiotics: Seven of 18 patients with prior sulfamethoxazole allergy reacted to oral challenge with sulfadiazine, and 4 of 9 patients with prior allergy with sulfamethoxazole reacted to challenge with sulfamethazine. All 28 patients in the study were challenged with furosemide (a sulfa nonantibiotic) with no allergic reactions.

Dr. Brian Strom and colleagues conducted a retrospective cohort study using general practice research database, looking at risk of allergic reactions within 30 days of receipt of a sulfonamide nonantibiotic (N. Engl. J. Med. 2003;349:1628-35).

Patients who had a history of prior hypersensitivity to a sulfonamide antibiotic were compared with those with no previous history of allergy. Analyses were also performed in patients with a prior penicillin allergy. A total of 969 patients who had an allergic reaction after a sulfonamide antibiotic were evaluated.

Of those patients, 9.1% had an allergic reaction after receiving a sulfonamide nonantibiotic. In those patients without a sulfa antibiotic allergy, only 1.6% had a reaction to a sulfa nonantibiotic. Interestingly, in patients with a prior history of penicillin reaction, 14.6% had an allergic reaction when receiving a sulfa nonantibiotic.

Patients with a history of sulfa allergy to a sulfa antibiotic are more likely to have a reaction to a sulfa nonantibiotic than are those without a previous allergy history. But this appears to be due to overall increased reactiveness and not a cross reactivity, because those with history of penicillin allergy had an even higher allergy rate to sulfa nonantibiotics than did patients with a prior sulfa allergy.

 

Dr. Paauw is professor of medicine in the division of general internal medicine at the University of Washington, Seattle, and he serves as third-year medical student clerkship director at the University of Washington. Contact Dr. Paauw at dpaauw@uw.edu.

Publications
Publications
Topics
Article Type
Display Headline
Myth of the Month: Why can’t I give my patient hydrochlorothiazide?
Display Headline
Myth of the Month: Why can’t I give my patient hydrochlorothiazide?
Legacy Keywords
sulfa drug, drug allergy, hydrochlorothazide, high blood pressure
Legacy Keywords
sulfa drug, drug allergy, hydrochlorothazide, high blood pressure
Sections
Disallow All Ads